LSAT and Law School Admissions Forum

Get expert LSAT preparation and law school admissions advice from PowerScore Test Preparation.

 LAM
  • Posts: 41
  • Joined: Dec 15, 2016
|
#34118
Can someone explain this to me? Each answer looks like it is out of left field. Answer choice C is asinine in my opinion because there is no assumption that dams on an existing river create land that are permanently under water. Please help. Thanks in advance.
 Francis O'Rourke
PowerScore Staff
  • PowerScore Staff
  • Posts: 471
  • Joined: Mar 10, 2017
|
#34161
You're right that dams often form lakes on land that was once dry, but not always. However, Answer choice (C) does not state with certainty that additional land will be submerged after completing the dam.
The current yearly output, if any, from Rochelle's land that will be permanently under water once the dam is completed...
The phrase "if any" explicitly tells us that absolutely zero farmland could be submerged after the dam is completed. It does say that there will be land that will be permanently under water; of course this land may have simply been the land that was already under the river, as in the case of a narrow gorge.

Let me know if you need any other help with this question :)
 yrivers
  • Posts: 68
  • Joined: Mar 15, 2017
|
#34576
Hi,

For question #11, I am able to see why all the other answers are wrong (a, b, d, e) but I'm still not persuaded that C is the correct answer. Specifically, I'm confused by "...will at least be matched by additional yearly output from Rochelle's remaining land." in answer C.

Thanks,
Yaesul
 Francis O'Rourke
PowerScore Staff
  • PowerScore Staff
  • Posts: 471
  • Joined: Mar 10, 2017
|
#34598
Hi Yaesul,

The statement you mentioned - current output... will at least be matched by additional yearly output from the remaining land - is perfectly constructed for us to use the Assumption Negation technique on.

Remember that if the question asks for an answer that must be true, or is required, or is one which the argument depends on, we can apply this technique. Since the conclusion requires the answer choice, the contra-positive tells us that if the answer choice is negated, then the conclusion must be false.

First negate this answer choice. Once you have this negated statement, look back at Betty's argument. Is her conclusion possible if we assume this negated answer?

Let us know if this helps or if you have further questions :-D
 yrivers
  • Posts: 68
  • Joined: Mar 15, 2017
|
#34600
Thank you, Francis!

I tried negating, and to me, the conclusion still holds. Why does the additional yearly output HAVE TO (caps added) at least match the currently yearly output that would be permanently under water? Even they didn't match, the agricultural output could stabilize once the dam is completed.
 Francis O'Rourke
PowerScore Staff
  • PowerScore Staff
  • Posts: 471
  • Joined: Mar 10, 2017
|
#34716
There's a very slight distinction in the stimulus you might be missing. Betty claims that the output will stabilize at its present level, meaning at the same level as the output before the dam. That's why we'll need it to at least match the output that is lost.

Get the most out of your LSAT Prep Plus subscription.

Analyze and track your performance with our Testing and Analytics Package.